musicprincess3
musicprincess3
03-11-2017
Mathematics
contestada
$8.50 shorts; 6.5% markup
Respuesta :
Jasmineeecx
Jasmineeecx
03-11-2017
Multiply
$9.50 by 1.065.
So, therefore the answer would be
$9.05 or 9.0525
Hope this helps, Good luck! let me know if you need me to explain it more (:
Answer Link
VER TODAS LAS RESPUESTAS ( 13+ )
Otras preguntas
find three numbers in dip, such that their sum is 60 and the last one is three times the first
Avoidant Restrictive Food Intake Disorder is characterized byA. distorted body image and an extreme limitation of calorie intakeB. limiting types and amounts
A 21-year-old woman presents to the emergency department with fevers, headache, neck stiffness, and mild confusion over the past several days. Her temperature i
!!!HELP ME!!! ILL GIVE YOU BRAINLIEST!!!! Celine has earned the following homework scores in her history class: 5, 0, 4, 8, 10, 9, 4, 6, 2. There is one homewor
1.Waheeda mixes water with some lemon juice to make lemonade. Write an equation to represent how much lemon juice is needed when Waheeda uses 18 ounces of water
PLS HELP ME ON THIS QUESTION I WILL MARK YOU AS BRAINLIEST IF YOU KNOW THE ANSWER!! Find the interquartile range of the data set that consists of 3, 11, 4, 3, 1
Does anyone know what topic this is ?
Marquice and Tim was selling tickets for the upcoming basketball game Marquice sol 25% more than 10 if they sold 45 tickets total how many tickets did each of t
tan theta is equal to 2- cos theta find the value of sin theta
what do you mean by Transformation?